LSAT and Law School Admissions Forum

Get expert LSAT preparation and law school admissions advice from PowerScore Test Preparation.

User avatar
 Dave Killoran
PowerScore Staff
  • PowerScore Staff
  • Posts: 5852
  • Joined: Mar 25, 2011
|
#88294
Complete Question Explanation
(The complete setup for this game can be found here: lsat/viewtopic.php?f=177&p=88290#p88290)

The correct answer choice is (E).

The condition in the question stem creates an M :longline: H sequence. If M cannot then be on the third-place team, then only N and O remain for the third-place team, and M must be on the first- or second-place team:

G4-Q22-d1.png

Of course, from the third rule, M and P cannot be on the same team, and so P must be on the team M is not on, creating an M/P dual-option:

G4-Q22-d2.png

The only remaining member—S—must then be on the first-place team:

G4-Q22-d3.png

If S is on the first-place team, then G must be the first-place team, leaving F and H to place second and third, not necessarily in that order:

G4-Q22-d4.png

Although the diagram is complete, the placement of M does still affect H. If M is on the second-place team, then H must be the third-place team. If M is on the first-place team, H can be the second- or third-place team.

From the diagram we can deduce that P can be the first-place team, and thus answer choice (E) is correct. All of the other answers are proven impossible by the diagram on the previous page.
You do not have the required permissions to view the files attached to this post.
 BryanEkller
  • Posts: 1
  • Joined: Nov 12, 2018
|
#60372
I don't see what is limiting this game. I have 4 hypotheticals and can't seem to find a good way to attack this question. Any advice?
 Claire Horan
PowerScore Staff
  • PowerScore Staff
  • Posts: 408
  • Joined: Apr 18, 2016
|
#60636
Hi Bryan,

This question can actually be solved efficiently without using templates. You might have thought to do templates for the possible rankings of the teams (GFH, GHF, and FGH), but that won't get you very far because most of the rules are focused on the individual team members.

Your diagram should look like the following, with MNOPST placed in the slots. Use FGH to write the rules and make the inferences, but the names of the high schools do not need to go on the diagram.


----       ----       ----


----       ----       ----
1       2       3

Approach this question by adding the implications of the local rule to your diagram, drawing the inferences, and finding the only answer choice that matches your local diagram.

From the global rules, we know that S and P cannot be on the third-ranked team. The local rule tells us that M can also not be on the third-ranked team. That leaves only O and N, so they must both be on the third-ranked team. M and P can't be on the same team, so one of them is on the first-ranked team and the other is on the second-ranked team. That leaves S, which must go in the only slot left, on the first-ranked team.

For this question, your local diagram ends up like this:

M/P       M/P       O/N
----       ----       ----

S       T       O/N
----       ----       ----
1       2       3

Good luck with your studies!

Get the most out of your LSAT Prep Plus subscription.

Analyze and track your performance with our Testing and Analytics Package.